เมื่อทฤษฎีบทขีด จำกัด กลางและกฎจำนวนมากไม่เห็นด้วย


19

นี่เป็นการจำลองแบบของคำถามที่ฉันพบที่ math.seซึ่งไม่ได้รับคำตอบที่ฉันหวังไว้

ปล่อยเป็นลำดับของตัวแปรสุ่มแบบกระจายที่เหมือนกันโดยมีและ . E [ X i ] = 1{Xi}iNE[Xi]=1V[Xi]=1

พิจารณาการประเมินผลของ

limnP(1ni=1nXin)

การแสดงออกนี้จะต้องมีการจัดการตั้งแต่นั้นมาทั้งสองด้านของเหตุการณ์ความไม่เท่าเทียมมีแนวโน้มที่จะไม่มีที่สิ้นสุด

A) การทดลองใช้งานระบบย่อย

ก่อนพิจารณาคำสั่งที่ จำกัด ให้ลบnจากทั้งสองด้าน:

limnP(1ni=1nXinnn)=limnP(1ni=1n(Xi1)0)=Φ(0)=12

ความเท่าเทียมกันครั้งสุดท้ายโดย CLT โดยที่Φ()เป็นฟังก์ชันการแจกแจงแบบปกติมาตรฐาน

B) คูณทวีคูณ

ทวีคูณทั้งสองข้างด้วย1/n

limnP(1n1ni=1nXi1nn)=limnP(1ni=1nXi1)

=limnP(X¯n1)=limnFX¯n(1)=1

โดยที่FX¯n()เป็นฟังก์ชันการแจกแจงของค่าเฉลี่ยตัวอย่างX¯nซึ่ง LLN มาบรรจบกันในความน่าจะเป็น (และในการกระจาย) ไปยังค่าคงที่1ดังนั้นความเท่าเทียมกันครั้งสุดท้าย

ดังนั้นเราจึงได้ผลลัพธ์ที่ขัดแย้งกัน อันไหนที่ถูก? และทำไมคนอื่นผิด


1
@JuhoKokkala แน่นอนนี่คือmath.stackexchange.com/q/2830304/87400เพียงแค่เพิกเฉยต่อความผิดพลาดของ OP ที่นั่น
Alecos Papadopoulos

2
ฉันคิดว่าปัญหาอยู่ในคำสั่งที่สองที่เรียกใช้
LLN

3
ฉันตามคุณไปจนถึงความเท่าเทียมสุดท้าย มันเป็นเรื่องที่ไม่ถูกต้องอย่างชัดเจนเพราะเราจะคาดหวังประมาณขนาดใหญ่และดังนั้นจึงขีด จำกัด ไม่ควรเท่ากับ เป็นเหตุผลตั้งใจของมันคืออะไร มันไม่ใช่คำแถลงของกฎหมายใด ๆ ที่มีจำนวนมากที่ฉันรู้ P(X¯n1)1/2n1.
whuber

1
@whuber คาดคะเนว่าน่าจะเป็นทั้งหมดสำหรับเข้มข้นเฉลี่ยตัวอย่างค่าที่1หากนี่เป็นสิ่งที่ผิดฉันเชื่อว่ามันเป็นสิ่งสำคัญสำหรับความผิดพลาดที่จะอธิบายรายละเอียดในคำตอบนั่นคือจุดประสงค์ของคำถามนี้ 1
Alecos Papadopoulos

2
Alecos, ความกังวลของฉันไม่ได้ว่าขั้นตอนสุดท้ายนั้นผิดหรือเปล่า: มันเกี่ยวข้องกับเหตุผลของคุณในการสร้างมันขึ้นมา นั่นไม่ใช่คำถามทั้งหมดหรือเปล่า? ฉันยังไม่ได้อ่านอะไรจากคุณให้เหตุผลเหล่านั้นและฉันก็ลังเลที่จะคาดเดาสิ่งที่พวกเขาอาจจะ แม้ว่าคุณจะอ้างถึง "LLN" แต่ฉันเชื่อว่าการแก้ไขปัญหาของคุณน่าจะอยู่ที่การอธิบายสิ่งที่คุณเข้าใจว่า "LLN" เพื่อยืนยัน
whuber

คำตอบ:


15

ข้อผิดพลาดที่นี่อาจเป็นไปได้ในความเป็นจริงต่อไปนี้: การบรรจบกันในการจัดจำหน่ายโดยปริยายสันนิษฐานว่าลู่ไปที่จุดของความต่อเนื่องของ(x) การกระจายขีด จำกัด เป็นของตัวแปรสุ่มคงมีต่อเนื่องกระโดดที่ , ด้วยเหตุนี้มันไม่ถูกต้องที่จะสรุปว่าลู่ CDF เพื่อ 1 F ( x ) F ( x ) x = 1 F ( x ) = 1Fn(x)F(x) F(x)x=1F(x)=1


1
วิธีที่เรานิยามการลู่เข้าหากันนั้นไม่ได้กีดกันความเป็นไปได้ของการบรรจบกัน ณ จุดที่ความไม่ต่อเนื่อง - มันไม่ต้องการมัน
Alecos Papadopoulos

1
แต่ถ้าการบรรจบกันในการแจกแจงไม่ต้องการเพื่อมาบรรจบกันที่ความเท่าเทียมกันสุดท้ายในคำถามนั้นมาจากอะไร? F ( 1 )Fn(1)F(1)
Juho Kokkala

1
@Juho มันไม่ได้ขึ้นอยู่กับอะไรเลย - นั่นเป็นประเด็นสำคัญของเรื่องนี้ ไม่มีทฤษฎีบทใดที่อนุญาตให้เราสร้างสมการสุดท้ายในคำถาม
whuber

1
@AlecosPapadopoulos: ฉันไม่เคยบอกว่ามันไม่ได้ยกเว้นความเป็นไปได้ ฉันบอกโดยปริยายว่าคุณต้องแสดงให้เห็นถึงความเท่าเทียมกันสุดท้ายที่เกินกว่าสิ่งที่คุณได้รับจากการลู่เข้าหากัน ตัวอย่างเช่นถ้าคือ Bernoulli มันจะเป็นจริง Xn
Alex R.

11

สำหรับตัวแปรสุ่มของ iidด้วย define ตอนนี้ CLT กล่าวว่าสำหรับทุกการแก้ไขจำนวนจริง ,(Z-1) OP ใช้ CLT เพื่อประเมิน E [ X i ] = var ( X i ) = 1 Z nXiE[Xi]=var(Xi)=1 zlimnFZn(z)=Φ(z-1)LimnP(Zn1

Zn=1ni=1nXi,Yn=1ni=1nXi.
zlimnFZn(z)=Φ(z1)
limnP(Zn1n)=Φ(0)=12.

เช่นเดียวกับคำตอบอื่น ๆ รวมถึงความคิดเห็นต่างๆเกี่ยวกับคำถามของ OP ได้ชี้ให้เห็นว่ามันเป็นการประเมินของที่สงสัย พิจารณากรณีพิเศษเมื่อ IIDเป็นตัวแปรสุ่มต่อเนื่องการที่ค่าและมีความน่าจะเท่ากับ12 ตอนนี้สามารถใช้ในทุกจำนวนเต็มคู่ค่าในและดังนั้นเมื่อเป็นเลขคี่, ไม่สามารถใช้กับค่าและด้วยเหตุนี้ไม่สามารถใช้กับค่าX i 0 2 1limnP(Yn1)Xi02 n12[0,2n]n n ฉัน= 1 XฉันnYn=1i=1nXi[0,2n]ni=1nXin1Yn1P(Yn1)=FYn(1)1Yn=1ni=1nXi 1. นอกจากนี้เนื่องจากการกระจายของสมมาตรประมาณเรามีที่ มีค่าเมื่อใดก็ตามที่เป็นเลขคี่ ดังนั้นลำดับของตัวเลข มีsubsequence ซึ่งในแง่ทั้งหมดที่มีค่า12 บนมืออื่น ๆ ที่subsequence จะมาบรรจบกันไป1ดังนั้นYn1P(Yn1)=FYn(1) nP(Y11),P(Y21),,P(12nP ( Y 11 ) , P ( Y 31 ) , , P ( Y 2 k - 11 ) , 1

P(Y11),P(Y21),,P(Yn1),
P(Y11),P(Y31),,P(Y2k11),
P(Y21),P(Y41),...,P(Y2k1),...1ลิมn→การP(Yn1)P(Yn1)12
P(Y21),P(Y41),,P(Y2k1),
1limnP(Yn1)ไม่มีอยู่จริงและการเรียกร้องการบรรจบกันของถึง 1 ต้องดูอย่างสงสัยมากP(Yn1)

8

ผลลัพธ์แรกของคุณคือผลลัพธ์ที่ถูกต้อง ข้อผิดพลาดของคุณเกิดขึ้นในส่วนที่สองในคำสั่งที่ผิดพลาดต่อไปนี้:

limnFX¯n(1)=1.

ข้อความนี้เป็นเท็จ (ทางด้านขวามือควรเป็น ) และไม่เป็นไปตามกฎหมายของคนจำนวนมากที่ถูกกล่าวหา กฎหมายอ่อนแอของคนจำนวนมาก (ซึ่งคุณเรียกใช้) กล่าวว่า:12

limnP(|X¯n1|ε)=1for all ε>0.

สำหรับทั้งหมดเงื่อนไขครอบคลุมค่าบางอย่างที่และค่านิยมบางอย่างที่1 ดังนั้นมันไม่เป็นไปตามจาก LLN ที่1| ˉ X n - 1 | ε ˉ X n1 ˉ X n > 1 ลิมn →การ P ( ˉ X n1 ) = 1ε>0|X¯n1|εX¯n1X¯n>1limnP(X¯n1)=1


1
ผลลัพธ์ (ผิดพลาดจริง ๆ ) มาจากความหมาย "การลู่เข้าในความน่าจะเป็นหมายถึงการลู่เข้าหากันในการกระจาย" คำถามไม่ได้ระบุว่าการยืนยันมาโดยตรงจาก LLN
Alecos Papadopoulos

@AlcosPapadopoulos: การบรรจบกันของความน่าจะเป็นการรวมตัวกันในการกระจาย อีกครั้งการรวมกันในการกระจายจะต้องเฉพาะที่จุดต่อเนื่อง แต่บางทีคุณอาจหมายถึงการบรรจบกันในความน่าจะไม่ได้หมายถึงpointwiseบรรจบกันของการกระจาย
Alex R.

@AlexR ฉันไม่แน่ใจว่าคำคัดค้านของคุณอยู่ที่ใด ฉันเชื่อว่าปัญหานี้ครอบคลุมในคำตอบของฉันเอง
Alecos Papadopoulos

3

การลู่เข้าในความน่าจะเป็นเป็นนัยของการลู่เข้าหากันในการกระจาย แต่ ... การกระจายแบบไหน ถ้าการ จำกัด การกระจายมีความไม่ต่อเนื่องของการกระโดดการ จำกัด จะไม่ชัดเจน (เนื่องจากมีค่าหลายค่าที่ไม่ต่อเนื่อง)

ที่เป็นฟังก์ชั่นการกระจายของค่าเฉลี่ยตัวอย่างซึ่งโดยลู่ LLN ในความน่าจะเป็น (และยังอยู่ในการจัดจำหน่าย) เพื่อคงที่ ,ˉ X n 1FX¯n()X¯n1

สิ่งนี้ไม่ถูกต้องและยังง่ายที่จะแสดงว่าไม่ถูกต้อง (แตกต่างจากความขัดแย้งระหว่าง CLT และ LLN) การ จำกัด การกระจาย (ซึ่งสามารถเห็นได้ว่าเป็นข้อ จำกัด สำหรับลำดับของตัวแปรการกระจายแบบปกติ) ควรเป็น:

FX¯(x)={0for x<10.5for x=11for x>1

สำหรับฟังก์ชั่นนี้คุณมีสิ่งนั้นและทุกๆความแตกต่าง สำหรับขนาดใหญ่พอnสิ่งนี้จะล้มเหลวหากแทนx | F ˉ X n ( x ) - F ˉ X ( x ) | < ϵϵ>0x|FX¯n(x)FX¯(x)|<ϵnFX¯(1)=1FX¯(1)=0.5


ขีด จำกัด ของการแจกแจงแบบปกติ

อาจเป็นประโยชน์ในการเขียนผลรวมที่ใช้เพื่อเรียกใช้กฎหมายจำนวนมากอย่างชัดเจน

X¯n=1ni=1nXiN(1,1n)

ขีด จำกัดสำหรับนั้นเทียบเท่ากับฟังก์ชัน Dirac Delta เมื่อมันถูกแทนด้วยขีด จำกัด ของการแจกแจงแบบปกติที่มีค่าความแปรปรวนเป็นศูนย์nX^n

การใช้นิพจน์นั้นง่ายกว่าที่จะเห็นสิ่งที่เกิดขึ้นภายใต้ประทุนแทนที่จะใช้กฎหมายสำเร็จรูปของ CLT an LLN ซึ่งปิดบังเหตุผลที่อยู่เบื้องหลังกฎหมาย


การลู่เข้าในความน่าจะเป็น

กฎจำนวนมากจะช่วยให้คุณ 'ลู่เข้าหาความน่าจะเป็น'

limnP(|X¯n1|>ϵ)=0

ด้วยϵ>0

คำสั่งที่เทียบเท่าสามารถสร้างขึ้นมาสำหรับทฤษฎีบทขีด จำกัด กลางที่มี limnP(|1n(Xi1)|>ϵn)=0

มันผิดที่จะระบุว่าสิ่งนี้หมายถึง

limnP(|X¯n1|>0)=0

เป็นเรื่องที่ดีน้อยกว่าที่คำถามนี้ถูกโพสต์ข้ามเร็ว (สับสน แต่น่าสนใจที่จะเห็นการอภิปราย / วิธีการทางคณิตศาสตร์ที่แตกต่างกันและสถิติไม่ได้เลวร้ายเกินไป) คำตอบโดยไมเคิลฮาร์ดี้ในข้อเสนอที่คณิตศาสตร์ stackexchange กับมันได้อย่างมีประสิทธิภาพมากในแง่ของกฎหมายที่แข็งแกร่งของตัวเลขขนาดใหญ่ (หลักการเดียวกับคำตอบที่ได้รับการยอมรับจาก drhab ในคำถามโพสต์ข้ามและดิลลิปได้ที่นี่) เราเกือบแน่ใจว่าลำดับเป็น 1 แต่นี่ไม่ได้หมายความว่าX¯1,X¯2,X¯3,...X¯nlimnP(X¯n=1)จะเท่ากับ 1 (หรืออาจไม่มีอยู่ตามที่ Dilip แสดง) ตัวอย่างของลูกเต๋าในคอมเม้นท์ของ Tomasz แสดงให้เห็นอย่างชัดเจนจากมุมที่แตกต่าง (แทนที่จะเป็นขีด จำกัด ที่ไม่มีอยู่ขีด จำกัด จะเป็นศูนย์) ค่าเฉลี่ยของลำดับของการทอยลูกเต๋าจะมาบรรจบกับค่าเฉลี่ยของลูกเต๋า แต่ความน่าจะเป็นที่จะเท่ากับนี้จะเท่ากับศูนย์


ฟังก์ชันขั้นตอน Heaviside และฟังก์ชัน Dirac delta

CDF ของ มีดังต่อไปนี้:X¯n

FX¯n(x)=12(1+erfx12/n)

ด้วยหากคุณต้องการ (เกี่ยวข้องกับฟังก์ชั่นขั้นตอน Heavisideส่วนประกอบของฟังก์ชัน Dirac delta เมื่อดูเป็นขีด จำกัด ของ การแจกแจงแบบปกติ)limnFX¯n(1)=0.5


ฉันเชื่อว่ามุมมองนี้จะช่วยแก้ปัญหาของคุณอย่างสังหรณ์ใจเกี่ยวกับ 'แสดงว่ามันผิด' หรืออย่างน้อยก็แสดงให้เห็นว่าคำถามเกี่ยวกับการทำความเข้าใจสาเหตุของความขัดแย้งของ CLT และ LLN นี้เทียบเท่ากับคำถามของการทำความเข้าใจ หรือลำดับของการแจกแจงปกติที่มีค่าความแปรปรวนลดลงเป็นศูนย์


2
การ จำกัด การกระจายของคุณนั้นไม่ใช่การแจกจ่ายเลย CDF ต้องถูกต้องอย่างต่อเนื่องในขณะที่มันได้อย่างชัดเจนไม่ได้อยู่ที่2/1 x=1/2
Alex R.

ความต่อเนื่องทางด้านขวาดูเหมือนว่าจำเป็นสำหรับทุกคนเรามีเป็นเหตุการณ์ซ้อนกันเราควรมีแต่สิ่งนี้เป็นจริงสำหรับกรณีของเราและที่จับคืออะไร? ความต่อเนื่องที่ถูกต้องนี้จำเป็นหรือไม่ขึ้นอยู่กับสัจพจน์ความน่าจะเป็นหรือเป็นเพียงแบบแผนที่ CDF ใช้งานได้สำหรับกรณีทั่วไปส่วนใหญ่? alimnFX(a+1n)=FX(a)Xa+1n
limnFX(a+1n)=limnP(Xa+1n)=P(limnXa+1n)=P(Xa)=FX(a)
Sextus Empiricus

@Martin Weterings: นี่เป็นที่มาของมันอย่างแม่นยำ การวัดที่ถูกต้องใด ๆจะต้องเป็นไปตามผลลัพธ์ของความน่าเบื่อหน่าย มันเป็นผลมาจากขอบเขตของพร้อมกับความสามารถนับได้ โดยทั่วไปฟังก์ชั่นคือ CDF (เช่นสอดคล้องกับการแจกแจงผ่าน iffถูกต้องต่อเนื่องพร้อมกับ monotonic และมีขีด จำกัด ที่เหลือ 0 ขีด จำกัด ที่ถูกต้อง 1PPF(x)PF(b)F(a)=P(a<Xb)F
อเล็กซ์อาร์

2

ฉันเชื่อว่ามันควรจะชัดเจนตอนนี้ว่า "วิธีการ CLT" ให้คำตอบที่ถูกต้อง

มาหาจุดที่ "วิธีการ LLN" ผิดพลาด

เริ่มต้นด้วยงบ จำกัด ก็เป็นที่ชัดเจนแล้วว่าเราสามารถทำได้เท่ากันทั้งลบจากทั้งสองฝ่ายหรือ multliply ทั้งสองข้างด้วย{n} เราได้รับn1/n

P(1ni=1nXin)=P(1ni=1n(Xi1)0)=P(1ni=1nXi1)

ดังนั้นหากมีขีด จำกัด อยู่ก็จะเหมือนกัน การตั้งค่าเรามีโดยใช้ฟังก์ชันการกระจายZn=1ni=1n(Xi1)

P(1ni=1nXin)=FZn(0)=FX¯n(1)

... และมันเป็นความจริงที่1/2limnFZn(0)=Φ(0)=1/2

ความคิดใน "LLN เข้าใกล้" ไปดังนี้: "เรารู้จาก LLN ที่ลู่เข้าในความน่าจะเป็นค่าคงที่และเราก็รู้ว่า" การลู่เข้าในความน่าจะเป็นหมายถึงการบรรจบกันของการกระจาย "ดังนั้น ลู่ ในการกระจายให้คงที่ " ถึงที่นี่เราถูกต้อง จากนั้นเราระบุว่า: "ดังนั้นการ จำกัด ความน่าจะเป็นสำหรับจะได้รับจากฟังก์ชั่นการกระจายของค่าคงที่ที่ตัวแปรสุ่ม",X¯nX¯n
X¯n1

F1(x)={1x10x<1F1(1)=1

... ดังนั้น ...limnFX¯n(1)=F1(1)=1

... และเราก็ทำผิดพลาดของเรา ทำไม? เพราะเป็น@AlexR คำตอบตั้งข้อสังเกต "การลู่เข้าในการกระจาย" ครอบคลุมเฉพาะจุดต่อเนื่องของฟังก์ชันการ จำกัด การกระจาย และเป็นจุดของความไม่ต่อเนื่องสำหรับF_1ซึ่งหมายความว่าอาจเท่ากับแต่มันอาจจะไม่ได้โดยไม่ปฏิเสธ "การบรรจบกันในการกระจายให้คงที่" ความหมายของ LLN .1F1limnFX¯n(1) F1(1)

และเนื่องจากจากวิธีการของ CLT เราจึงรู้ว่าค่าของขีด จำกัด ต้องเป็น ( ) ผมไม่ทราบวิธีที่จะพิสูจน์โดยตรงว่า1/21/2limnFX¯n(1)=1/2

พวกเราเรียนรู้สิ่งใหม่หรือไม่?

ฉันทำ. LLN อ้างว่า

limnP(|X¯n1|ε)=1for all ε>0

limn[P(1ε<X¯n1)+P(1<X¯n1+ε)]=1

limn[P(X¯n1)+P(1<X¯n1+ε)]=1

LLN ไม่ได้บอกว่าความน่าจะเป็นที่จัดสรรในช่วงอย่างไร สิ่งที่ฉันได้เรียนรู้คือในผลลัพธ์ของการลู่ในชั้นนี้ความน่าจะเป็นอยู่ที่ขีด จำกัด ที่จัดสรรให้เท่ากันทั้งสองด้านของจุดกึ่งกลางของช่วงยุบ (1ε,1+ε)

ข้อความทั่วไปที่นี่คือสมมติว่า

Xnpθ,h(n)(Xnθ)dD(0,V)

ที่คือ RV บางคนที่มีฟังก์ชั่นการกระจายF_Dแล้วก็DFD

limnP[Xnθ]=limnP[h(n)(Xnθ)0]=FD(0)

... ซึ่งอาจไม่เท่ากับ (ฟังก์ชันการแจกแจงของค่าคงที่ rv)Fθ(0)

ยิ่งไปกว่านั้นนี่คือตัวอย่างที่ดีว่าเมื่อฟังก์ชันการกระจายของตัวแปรสุ่มแบบ จำกัด มีความไม่ต่อเนื่องจากนั้น "การบรรจบกันในการกระจายไปยังตัวแปรแบบสุ่ม" อาจอธิบายสถานการณ์ที่ "การ จำกัด การกระจาย" อาจไม่เห็นด้วยกับ "การกระจายของการ จำกัด ตัวแปรสุ่ม "ที่จุดไม่ต่อเนื่อง การพูดอย่างเคร่งครัดการแจกแจง จำกัด สำหรับจุดต่อเนื่องนั้นเป็นของตัวแปรสุ่มคงที่ สำหรับคะแนนความไม่ต่อเนื่องเราอาจคำนวณความน่าจะเป็นที่ จำกัด ได้ในฐานะหน่วยงาน "แยก"


มุมมอง 'บทเรียนที่เรียนรู้' นั้นน่าสนใจและนี่เป็นตัวอย่างที่ดีไม่ยากเกินไปสำหรับการสอนเชิงปฏิบัติ แม้ว่าฉันจะสงสัยว่าแอปพลิเคชั่นที่ใช้งานได้จริงชนิดใดที่ความคิดนี้เกี่ยวกับอนันต์นั้นเป็นเพราะในที่สุดในทางปฏิบัติn
Sextus Empiricus

@ Martijn Weterings Martijn แรงบันดาลใจที่นี่คือการศึกษาอย่างแน่นอนก) เพื่อเป็นการแจ้งเตือนถึงความไม่ต่อเนื่องแม้ในสถานการณ์ "แบน" เช่นการบรรจบกันเป็นค่าคงที่และโดยทั่วไป (พวกเขาทำลายการบรรจบกันอย่างสม่ำเสมอ) และ b) ผลลัพธ์เกี่ยวกับวิธีการจัดสรรความน่าจะเป็นที่น่าสนใจเมื่อลำดับที่มาบรรจบกันในความน่าจะเป็นคงที่ยังคงมีความแปรปรวนที่ไม่เป็นศูนย์
Alecos Papadopoulos

เราสามารถพูดได้ว่า CLT สมมติว่ามีอะไรบางอย่างเกี่ยวกับการลู่เข้ากับตัวแปรที่กระจายตัวแบบ จำกัด (ทำให้สามารถแสดงสิ่งต่าง ๆ เช่น ) แต่ LLN ช่วยให้เราสามารถพูดได้ว่าโดยการเพิ่มขนาดตัวอย่าง เป็นค่าเฉลี่ยจริง แต่สิ่งนี้ไม่ได้บอกว่าเราได้มาด้วยความน่าจะเป็นที่สูงขึ้น 'เท่ากับค่าเฉลี่ยตัวอย่าง' LLN หมายถึงค่าเฉลี่ยตัวอย่างนั้นเข้ามาใกล้ยิ่งขึ้นและใกล้เคียงกับค่า จำกัด แต่ไม่ใช่ (ที่มีความน่าจะเป็นสูงกว่า) เท่ากับ LLN พูดอะไรเกี่ยวกับF ( x )F(x)F(x)
Sextus Empiricus

ความคิดดั้งเดิมรอบ LLN ซึ่งตรงกันข้ามกับจริง (ดูเหตุผลของ Arbuthnot stats.stackexchange.com/questions/343268 ) "มองเห็นได้จากสิ่งที่ถูกกล่าวว่าด้วยจำนวนลูกเต๋าที่ดีมากล็อตของ A จะเล็กมาก ... จะมี แต่ส่วนเล็ก ๆ ของโอกาสที่เป็นไปได้ทั้งหมดที่เกิดขึ้นในเวลาที่กำหนด ควรมีจำนวนเพศชายและหญิงเท่ากัน "
Sextus Empiricus
โดยการใช้ไซต์ของเรา หมายความว่าคุณได้อ่านและทำความเข้าใจนโยบายคุกกี้และนโยบายความเป็นส่วนตัวของเราแล้ว
Licensed under cc by-sa 3.0 with attribution required.